next up previous contents
Nächste Seite: Approximation der Eulerschen Zahl Aufwärts: Ungleichungen Vorherige Seite: Ungleichungen   Inhalt


Bernoullische Ungleichung

Satz 1.5.1 (Bernoullische Ungleichung)   Es gilt

$\displaystyle (1+x)^n\geq 1+nx$    für $\displaystyle x\geq -2$ und $\displaystyle n \in \mathbb{N}\ .
$

Jakob Bernoulli, 1654-1705.

Anmerkung: Zum Beweis unterscheide man die Fälle:

$ -2 \leq x < -1$:
klar
$ -1 \leq x$:
Vollständige Induktion.

Satz 1.5.2  

  1. Für alle $ q \in (0,1)$ existiert ein $ C>0$ so, daß für alle $ n\in \mathbb{N}$ gilt:

    $\displaystyle q^n \leq \frac{C}{n}.
$

  2. Für alle $ q > 1$ existiert ein $ C>0$ so, daß für alle $ n\in \mathbb{N}$ gilt:

    $\displaystyle q^n \geq Cn.
$

Hinweis: (1) Man schreibe

$\displaystyle C :=\frac{q}{1-q} \quad\Leftrightarrow\quad \frac{1}{q} = 1 + \frac{1}{C}\ .
$

und benutze nun die Bernoullische Ungleichung.

Zur Abschätzung der Fakultäten $ n!$ ist es naheliegend die Faktoren durch ihren ,,Mittelwert`` $ \frac{n}{2}$ zu ersetzen.

Wir zeigen in den beiden folgenden Sätzen, daß für $ n=1,2,\dots$

$\displaystyle 3\left( \frac{n}{3} \right)^n \leq n! \leq 2\left( \frac{n}{2} \right)^n
$

gilt.

Lemma 1.5.3   Für alle $ n\in \mathbb{N}$ ist

$\displaystyle 2 \leq \left(1+\frac{1}{n} \right)^n .
$

Satz 1.5.4   Für $ n\in \mathbb{N}$ gilt:

$\displaystyle n!\leq 2\left(\frac{n}{2}\right)^n.
$

Beweis .

\fbox{$n=1$:}
nachrechnen!
\fbox{$n\Rightarrow n+1$:}

$\displaystyle {
(n+1)! = n!\,(n+1)}$
    $\displaystyle \leq 2\left(\frac{n}{2}\right)^n (n+1)$   (nach Induktionsvoraussetzung)  
    $\displaystyle = 2 \left(\frac{n+1}{2}\right)^n \left(\frac{n}{n+1}\right)^n (n+1)$  
    $\displaystyle = 2\left(\frac{n+1}{2}\right)^n
\frac{1}{\left(1+\frac{1}{n}\right)^{n}}\,(n+1)$  
    $\displaystyle \leq 2\left(\frac{n+1}{2} \right)^n \frac{1}{2}\, (n+1)$  
    $\displaystyle = 2\left(\frac{n+1}{2} \right)^{n+1}.$  

Lemma 1.5.5   Für alle $ n\in \mathbb{N}$ ist

$\displaystyle \left(1+\frac{1}{n} \right)^n < 3.
$

Beweis . Für $ k=1,\dots,n$ ist

$\displaystyle \binom{n}{k} \frac{1}{n^k}
=\frac{1}{k!} \prod_{l=0}^{k-1} \frac{n-l}{n}
\leq \frac{1}{k!} \leq \frac{1}{2^{k-1}}.
$

Also

$\displaystyle {
\left( 1+ \frac{1}{n} \right)^n = \sum_{k=0}^n \binom{n}{k}\frac{1}{n^k} }$
    $\displaystyle \leq 1 + \sum_{k=1}^n \frac{1}{2^{k-1}}$  
    $\displaystyle = 1 + \frac{ 1-(\frac{1}{2})^n } { 1-\frac{1}{2} } < 3.$  

Satz 1.5.6   Für alle $ n\in \mathbb{N}$ ist

$\displaystyle 3\left(\frac{n}{3} \right)^n \leq n!
$

Beweis .

\fbox{$n=1$:}
klar.
\fbox{$n\Rightarrow n+1$:}
Die Ungleichung gelte für ein $ n$, dann folgt:
$\displaystyle (n+1)! = (n+1)\,n!$ $\displaystyle \geq$ $\displaystyle (n+1)\, 3\,\left( \frac{n}{3}\right)^n$  
  $\displaystyle \geq$ $\displaystyle (n+1) \left(1+\frac{1}{n} \right)^n \left( \frac{n}{3}\right)^n$  
  $\displaystyle =$ $\displaystyle 3\, \frac{n+1}{3} \left(\frac{n+1}{n} \right) ^n \left(\frac{n}{3} \right)^n$  
  $\displaystyle =$ $\displaystyle 3 \left(\frac{n+1}{3} \right)^{n+1}$  


next up previous contents
Nächste Seite: Approximation der Eulerschen Zahl Aufwärts: Ungleichungen Vorherige Seite: Ungleichungen   Inhalt
Analysis1-A.Lambert 2001-02-09